Sunteți pe pagina 1din 13

Real Numbers

This Section is taken from the Book:

ISBN : 9789388919722
This book is available at all leading physical
book stores and online book stores.
To view complete books visit.

To download complete catalogue click


https://amzn.to/2GXTMyA or visit QR.
2

1 REAL NUMBERS
Chapter

CHAPTER AT A GLANCE

1. Euclid's Division Lemma : For given any two positive integers a and b,
there exist unique integers q and r satisfying
a = bq + r, 0 £ r < b
2. Lemma : A lemma is a proven statement used for proving another
statement.
3. Euclid's Division Algorithm : Euclid's division algorithm is a technique to
compute the Highest Common Factor (HCF) of two given positive integers.
To get HCF of two positive integers c and d, c > d following steps are to be
followed:
(i) Apply Euclid's division lemma to c and d to get whole numbers q and r
such that
c = dq + r, 0 £ r < d.
(ii) If r = 0, then d is HCF of c and d. If r ¹ 0, apply division lemma to d and r.
(iii) Continue the process till the remainder is zero. The divisor at this stage
will be the required HCF.
Note:
(i) Euclid's division lemma and algorithm are so closely interlinked that
people often call former as the division algorithm also.
(ii) Euclid's division algorithm is stated for only +ve integers but it can be
extended for all integers except zero.
4. Fundamental Theorem of Arithmetic : Every composite number can be
expressed (factorised) as a product of primes, and this factorisation is unique,
apart from the order in which the prime factors occur.
The prime factorisation of a natural number is unique, except for the order of
its factors. In general, given a composite number x, we factorise it as
x = p1p2p3 ........pn, where p1, p2, p3 ............, pn are primes and written in
ascending order, i.e.,
p1 £ p2 £ p3 £ ..........£ pn. If we combine the same primes, we will get powers
of primes.
3
For Example:
156

2 × 78 3 × 52 4 ×39 6 ×26

2 × 2 × 39 3 × 2 × 26 4 × 3 × 13 6 × 2 × 13

2 × 2 × 3 × 13 3 × 2 × 2 × 13 2 × 2 × 3 × 13 2 × 3 × 2 ×13

So, in each of the cases prime factors of 156 is 2 × 2 × 3 × 13


Hence, we can conclude that the prime factorisation of a number is unique.
Note:
(i) For two positive integers a, b
HCF (a, b) × LCM (a, b) = a × b
(ii) If p is a prime number and it divides a2, then p also, divides a where ‘a’
is the positive integer.
5. HCF & LCM of Three Numbers
p × q × r × HCF( p, q, r )
(i) LCM (p, q, r) =
HCF ( p, q) × HCF(q, r ) × HCF( p, r )
p × q × r × LCM( p, q, r )
(ii) HCF (p, q, r) =
LCM ( p, q) × LCM(q, r ) × LCM( p, r )
(iii) For three positive integers a, b and c
HCF (a, b, c) × LCM (a, b, c) ¹ a × b × c
6. Irrational Numbers : Number which is not a rational number whose
decimal expansion is non-terminating and non-repeating
Note:
(i) The sum or difference of a rational and an irrational number is irrational.
E.g., 2 + 2 is irrational, 2 - 3 is irrational.
(ii) The product and quotient of a non-zero rational and irrational number is
irrational.
2
E.g., 5 ´ 2 is irrational, is irrational.
3
7. Rational Numbers and Their Decimal Expansion
(i) If denominator of a rational number is of the form 2n 5m, where n, m are
non-negative integers then x has decimal expansion which terminates.
(ii) If decimal expansion of rational number terminates then its denominator
has prime factorisation of the form 2n 5m, where n, m are non-negative
integers.
(iii) If denominator of a rational number is not of the form 2n 5m, where n and
m are non-negative integers then the rational number has decimal
expansion which is non-terminating repeating.
4
Thus we conclude that the decimal expansion of every rational number
is either terminating or non-terminating repeating.
p
Regarding decimal expansion of rational number x = where p, q are co-prime
q
integers and q ¹ 0, we have
(i) x is a terminating decimal expansion if prime factorisation of q is of the form
2m 5n where m,n are non-negative integers.
(ii) If prime factorisation of q is not of the form 2m 5n then x is a non-terminating repeating
decimal expansion.

1.1
1. Use Euclid’s division algorithm to find the HCF of :
(i) 135 and 225 (ii) 196 and 38220 (iii) 867 and 255

Use Euclid's division algorithm, a = bq + r ; o £ r < b


where a, b, q and r are integer's. Here a = dividend and b = divisor
Sol. (i) Here, dividend = 225 and divisor = 135
\ 225 = 135 × 1 + 90
135 = 90 × 1 + 45
90 = 45 × 2 + 0
\ HCF (225, 135) = HCF (135 , 90) = HCF (90, 45) = 45
Hence, the HCF (225, 135) = 45
(ii) We have: Dividend = 38220 and Divisor = 196
38220 = 196 × 195 + 0
Hence, HCF (196, 38220) = 196
(iii) We have: Dividend = 867 and Divisor = 255
\ 867 = 255 × 3 + 102
255 = 102 × 2 + 51
102 = 51 × 2 + 0
Hence, HCF (255, 867) = 51
2. Show that any positive odd integer is of the form 6q + 1, or 6q + 3, or
6q + 5,where q is some integer.
Sol. Let a be positive odd integer.
By division algorithm,
a = 6 ´ q + r; 0£r <6
Here the possible integers can be:
a = 6q + 0 or 6q + 1 or 6q + 2 or 6q + 3 or 6q + 4 or 6q + 5.
5
We reject 6q + 0, 6q + 2, and 6q + 4 because they are even integers.
Hence, the possible form of a is 6q + 1, or 6q + 3, or 6q + 5.
3. An army contingent of 616 members is to march behind an army band of 32
members in a parade. The two groups are to march in the same number of
columns. What is the maximum number of columns in which they can march ?

Use Euclid's division algorithm to find the maximum number which can divide both
616 & 32.
Sol. Let n = number of columns required.
Now, we need to calculate maximum number of column in which both army
contingent and army band can march, i.e; we need to calculate highest number
which can divide both 616 & 32.
\ n = HCF (616, 32).
Using Euclid's Division Lemma,
\ 616 = 32 × 19 + 8
32 = 8 × 4 + 0
Hence, HCF (616, 32) = 8
Þ n= 8
Hence, the maximum number of the columns in which they can march = 8.
4. Using E.D.L show that the square of any positive integer is either of the form
3m or 3m + 1 for some integer m.
Sol. Let n be a positive integer.
Using E.D.L, we can write 'n' as n = 3q + r,0 £ r < 3
\ n can be of the form : 3q or 3q + 1 or 3q + 2.
If n = 3q Þ n2 = (3q)2 = 3 × 3q2 = 3m, where m = 3q2
If n = 3q + 1 Þ n2 = 3 (3q2 + 2q) + 1 = 3m + 1, where m = 3q2 + 2q
If n = 3q + 2 Þ n2 = 3 (3q2 + 4q + 1) + 1= 3m + 1, where m = 3q2 + 4q + 1
Thus, n2 can be either of the form 3m or 3m + 1.
5. Use Euclid’s division lemma to show that the cube of any positive integer is of
the form 9m, 9m + 1 or 9m + 8.
Sol. Let n be a positive integer.
Using E.D.L, we can write 'n' as : n = 3q + r, 0 £ r < 3
\ n can be of the form: 3q or 3q + 1 or 3q + 2
If n = 3q, n3 = 27 q3 = 9 ´ 3q3 = 9 m where m = 3q3
If n = 3q + 1 Þ n3 = (3q + 1)3
3 2
Þ n3 = 9{3q + 3q + q} + 1 Q ((a + b)3 = a3 + b3 + 3ab2 + 3a2b)
= 9m + 1 , where m = 3q3 + 3q2 + q
6
If n = 3q + 2 Þ n3 = (3q + 2)3 = 27q3 + 45q2 + 36q
Þ n = 9(3q3 + 6q2 + 4q) + 8 = 9m + 8, where m = 3q3 + 6q2 + 49
3

Similarly, n3 = 9m + 8
Þ n3 is either of the form 9m or 9m + 1 or 9m + 8.

1.2
1. Express each number as a product of its prime factors:
(i) 140 (ii) 156 (iii) 3825
(iv) 5005 (v) 7429
Sol. (i)
140

2 70

2 35

5 7

\ 140 = 2 × 2 × 5 × 7 = 22 × 5 × 7
(ii) We use the division method as shown below :
2 156
2 78
3 39
13 13
1
\ 156 = 2 × 2 × 3 × 13 = 22 × 3 × 13
7
(iii) Using division method, we have:
5 3825
5 765
3 153
3 51
17
\ 3825 = 3 × 3 × 5 × 5 × 17 = 32 × 52 × 17
(iv) We use the division method as shown below:
5 5005
7 1001
11 143
13 13
1
\ 5005 = 5 × 7 × 11 × 13
(v) We use the division method as shown below :
17 7429
19 437
23 23
1
\ 7429 = 17 × 19 × 23

Prime factors can be calculated either by Prime Factorisation Method or by Division


Method.

2. Find the LCM and HCF of the following pairs of integers and verify that
LCM × HCF = product of the two numbers.
(i) 26 and 91 (ii) 510 and 92 (iii) 336 and 54

HCF is the product of the smallest power of each common prime factor in the
number. LCM is the product of the greatest power of each prime factor involved in
the numbers.
Sol. (i) 26 = 2 × 13 and 91 = 7 × 13
HCF (26, 91) = 13 ,
LCM = (26, 91) = 2 × 7 × 13 = 182
LCM × HCF = 182 × 13 = 2366,
and 26 × 91 = 2366.
Therefore, LCM × HCF = 26 × 91 (Hence verified).
(ii) 510 = 2 × 3 × 5 × 17 and 92 = 2 × 2 × 23
\ LCM of 510 and 92 = 2 × 2 × 3 × 5 × 17 × 23 = 23460
8
and HCF of 510 and 92 = 2
\ LCM × HCF = 23460 × 2 = 46920 and 510 × 92 = 46920
\ LCM × HCF = 510 × 92 (Hence verified).
(iii) 336 = 2 × 2 × 2 × 2 × 3 × 7 and 54 = 2 × 3 × 3 × 3
\ LCM of 336 and 54 = 2 × 2 × 2 × 2 × 3 × 3 × 3 × 7 = 3024
and HCF of 336 and 54 = 2 × 3 = 6
Now, HCF × LCM = 3024 × 6 = 18144
and 336 × 54 = 18144
\HCF × LCM = 336 × 54 (Hence verified).
3. Find the LCM and HCF of the following integers by applying the prime
factorisation method.
(i) 12 , 15 and 21 (ii) 17 , 23 and 29 (iii) 8, 9 and 25
Sol. (i) First we write the prime factorisation of each of the given numbers.
12 = 22 × 3; 15 = 3 × 5 and 21 = 3 × 7
HCF (12, 15, 21) = 3
LCM (12, 15, 21) = 22 × 3 × 5 × 7 = 420
(ii) First we write the prime factorisation of each of the given numbers.
17 = 17; 23 = 23 and 29 = 29
\ LCM = 17 × 23 × 29 = 11339 and HCF = 1
(iii) First we write the prime factorisation of each of the given numbers.
8 = 2 × 2 × 2 = 23;9 = 3 × 3 = 32 and 25 = 5 × 5 = 52
\ LCM = 23 × 32 × 52 = 8 × 9 × 25 = 1800 and HCF = 1

(a) Every integer, n > 1 has a prime factor. If in is a prime number, then its prime
factor is n itself. If n is composite number, then it has prime factors less than
itself.
(b) LCM of two or more numbers having no common prime factor is equal to
their product. and HCF of two or more numbers having no common prime
factor is 1. As we have seen in Q 3 [(ii) and (iii)]

4. Given that HCF (306, 657) = 9, find LCM (306, 657).


Sol. We know that; HCF (a, b) × LCM (a, b) = a × b
Here; a = 306, b = 657 & HCF = 9.
Since, HCF (a, b) × LCM (a, b) = a × b
306 ´ 657
\ 9 × LCM = 306 × 657 Þ LCM = = 34 ´ 657 = 22338
9
\ LCM (306, 657) = 22338
5. Check whether 6n can end with the digit 0 for any natural number n.

Sol. Let if possible 6n ends with digit 0. And let 6n = 10 × q


n n
Þ 2n ´ 3n = 2 ´ 5 ´ q Þ 5 is a prime factor of 2 ´ 3
9
which is not possible because 2n ´ 3n can have only 2 and 3 as prime factors.
Hence, 6n cannot end with the digit 0 (zero) for any natural number n.
6. Explain why 7 × 11 × 13 + 13 and 7 × 6 × 5 × 4 × 3 × 2 × 1 + 5 are composite
numbers.

A composite number is a positive integer which is not prime (i.e; which has factors
other then 1 and itself).
Sol. (i) 7 × 11 × 13 + 13 = 13 × {7 × 11 + 1} = 13 × 78 = 2 × 3 × 132
Here, we get product of primes and this factorisation is unique. Hence,
this is a composite number.
(ii) 7 × 6 × 5 × 4 × 3 × 2 × 1 + 5 = 5 × {7 × 6 × 4 × 3 × 2 × 1 + 1}
= 5 × 1009
Here, we get product of primes and this factorisation is unique. Hence,
this is a composite number.
7. There is a circular path around a sports field. Sonia takes 18 minutes to
drive one round of the field, while Ravi takes 12 minutes for the same.
Suppose they both start at the same point and at the same time, and go
in the same direction. After how many minutes will they meet again at the
starting point?
Sol. Let 'n' minutes be the required time.
So, first time, they will meet again in 'n' minutes. Second time, they will meet
again in '2n' minutes & so on.
Þ n is the minimum time required i.e; for 18 minutes & 12, minutes, 'n' will be
the LCM of 18 & 12, i.e; lowest number which is the multiple of 18 & 12 both.
Here, 18 = 2 × 3 × 3 and 12 = 2 × 2 × 3
\ LCM (18, 12) = 2 × 2 × 3 × 3 = 36.
\ Ravi and Sonia will meet again at the starting point after 36 minutes.

1.3
1. Prove that 5 is irrational.

Use the method of contradiction, i.e., we will assume that 5 is not irrational or 5
is rational
10
Sol. Suppose 5 is not irrational,
p
i.e; 5 is rational & can be written as: 5= , where p & q are co-primes.
q
Þ 5q=p
Squaring both sides we get :
5 × q2 = p2 ...(i)
Þ 5 divides p (Q p & q are co-primes)
Þ p = 5 × p1 ; p1 is an integer. ...(ii)
Put the value of p from (ii) in (i), we get,
5 × q2 = (5 × p1)2 = 52 ´ p12 Þ q 2 = 5 ´ p12
Þ 5 divides q (Q p1 & q are co-primes)
Þ q = 5 ´ p2 ; p2 is an integer.. ...(iii)
From (ii) and (iii), we find 5 as a common factor of p and q. It contradicts that
p and q are co-primes and 5 is rational number..
Hence, 5 is an irrational number..

Imperfect squares are always irrational i.e, 2, 20, 30 etc.

2. Prove that 3 + 2 5 is irrational.


Sol. Suppose 3 + 2 5 is not irrational, i.e; 3 + 2 5 is rational & can be written as:
p
3+ 2 5 = where p & q are co-primes
q

p – 3q
Þ 2 5= (a rational number)
q

p – 3q
Þ 5= (a rational number) Þ 5 is a rational number..
2q
From Q 1, we get; 5 is irrational number, which contradicts our supposition.
Hence, 3 + 2 5 is irrational.
3. Prove that the following are irrationals :
1
(i) (ii) 7 5 (iii) 6+ 2
2
11
1 1 2 2
Sol. (i) = ´ = (rationalising)
2 2 2 2

2
If possible, let be rational.
2

Then 2 p , where p and q are integers and (q ¹ 0)


=
2 q
2p 2p
Þ = 2 Since 2, p and q are integers, we get is rational Þ 2 is
q q
rational. But we know that 2 is irrational, which contradicts our supposition.
2
\ our assumption that is rational, is wrong.
2
2 1
Þ is irrational or is irrational.
2 2
p p
(ii) 7 5= Where p and q are integers (q ¹ 0) Þ 5=
q 7q
Þ 5 is rational. It gives contradiction.
\ 7 5 is irrational.
p p – 6q
(iii) 6 + 2 = Where p and q are integers (q ¹ 0) Þ 2=
q q
Þ2 is a rational number. It gives contradictions.
\ 6 + 2 is irrational.

1.4
1. Without actually performing the long division, state whether the following
rational numbers will have a terminating decimal expansion or a
non-terminating repeating decimal expansion:
13 17 64 15
(i) (ii) (iii) (iv)
3125 8 455 1600
29 23 129 6
(v) (vi) 3 2 (vii) 2 7 5 (viii)
343 2 5 2 5 7 15
35 77
(ix) (x)
50 210
12
31 31 31
Sol. (i) = = .
5
3125 5 2 ´ 55
0

Since, q = 20 × 55 is in the form of 2m5n.


Hence decimal expansion is terminating.
17 17
(ii) =
8 2 ´ 50
3

Since, q = 23 × 50 is in the form of 2m5n. Hence, terminating.


64 64
(iii) = ;
455 5 ´ 7 ´ 13
Since, q = 5 × 7 × 13 is not in the form of 2m5n.
Hence, non-terminating repeating.
15 3 3
(iv) = =
1600 320 26 ´ 52
Since, q = 26 × 52 is in the form 2m5n. Therefore, terminating.
29 29 29
(v) = = ;
343 7 ´ 7 ´ 7 73
Since q = 7 × 7 × 7 is not in the form of 2m5n.
Therefore, non-terminating repeating.
23
(vi)
2352
Since, q = 23 × 52 is in the form of 2m5n. Therefore, terminating.
129
(vii) 2 7 5
2 5 7
Since, q = 225775 is not in the form of 2m5n.
Therefore, non-terminating repeating.
6 6
(viii) =
15 3 ´ 5
Since, q = 3 × 5 is not in the form of 2m5n.
Therefore, non-terminating repeating.
35 35
(ix) =
50 2 ´ 52
Since, q = 2 ×52 is in the form of 2m5n. Therefore, terminating
77 77
(x) =
210 2 ´ 3 ´ 5 ´ 7
Since, q = 2 × 3 × 5 × 7 is not in the form of 2m5n.
Therefore, non-terminating repeating.
13
2. Write down the decimal expansions of those rational numbers in Question 1
above which have terminating decimal expansion.

13 25 ´ 13 416
Sol. (i) = = = 0.00416
3125 25 ´ 55 105

17 17 ´ 53 17 ´ 53 2125
(ii) = = = 3 = 2.125
8 23 ´ 53 103 10

15 3 ´ 5 ´ 54 3 ´ 55 = 9375
(iv) = = = 0.009375
1600 26 ´ 52 ´ 54 10 6 106

23 23 ´ 5 115 115
(vi) 3 2
= 3 2
= 3
= = 0.115
2 ´5 2 ´5 ´5 10 1000

6 3´ 2 ´ 2 2 ´ 2
(viii) = = = 0.4
15 3 ´ 5 ´ 2 2 ´ 5
35 7 ´ 5 7
(ix) = = = 0.7
50 5 ´10 10
3. The following real numbers have decimal expansions as given below. In each
case, decide whether they are rational or not. If they are rational, and of the
p
form , what can you say about the prime factors of q?
q
(i) 43.123456789
(ii) 0.120120012000120000...
(iii) 43.123456789
Sol. (i) Rational: q = 109 = 29 × 59.
(ii) Non-rational.
(iii) Rational, prime factors of q will also have factors other than 2 or 5 because
the decimal expansion is non-terminating repeating.

S-ar putea să vă placă și